+0  
 
+1
697
5
avatar+15 

I got a quiz back and I didn't understand why I missed a question. The teacher explained it, something like "we read range from the bottom to the top," but I still don't understand it.

 

Here's the equation: f(x)=(2x3+5x2-3x)/(x2-x)

 

I got the range to be (-inf, 3.343] U [14.657, inf) using my graphing calculator.

 

She said that the range was (-inf, 3) U (3, 3.343] U [14.657, inf).

 

Now, don't get me wrong, I know there's a hole at (0, 3), but I thought that since there was another point where it crossed the y-axis, being (-1, 3), that 3 would be a valid value of the range.

 

Who's right and who's wrong? If my teacher is right can someone give me an explanation or a helpful video or something?

 

Thanks!

 Sep 25, 2019
 #1
avatar+128475 
+2

                  (2x^3 + 5x^2 - 3x)  

f(x) =           ______________   =      

                         x^2  -  x

 

 

x (2x^2 + 5x - 3)  

______________  =

 

   x  (x - 1)

 

 

2x^2 + 5x - 3

___________  =

       x  -  1

 

 

(2x - 1) ( x + 3)

____________

        x  - 1

 

We have a "hole"  at x  =  0     and a vertical asymptote at   x  =  1   

 

We  also have a "slant" asymptote   determined as follows :

 

                2x   +  7

x^2 - x   [ 2x^3 + 5x^2 - 3x  ]

               2x^3  - 2x^2

              ________________

                           7x^2  - 3x

                           7x^2  - 7x

                         __________

 

So.....the slant asymptote  is y  = 2x + 7

 

The range   is    (-inf, 3.343]  U  [ 14.657, inf)

 

Note that the graph crosses the line y =  3.....so......it definitely exists  when y = 3

 

See the graph here :  https://www.desmos.com/calculator/e5di4axkpw

 

 

 

 

cool cool cool

 Sep 25, 2019
edited by CPhill  Sep 25, 2019
 #2
avatar+15 
+2

Thank you so much! :)

You are the best!

RandomEpicGamer  Sep 25, 2019
 #3
avatar+128475 
+2

I made a slight mistake on the first posting.....there should be brackets next to 3.343  and 14.657

 

CORRECTED!!!

 

 

 

cool cool cool

CPhill  Sep 25, 2019
 #4
avatar+15 
+2

Okay! Thanks again!

RandomEpicGamer  Sep 25, 2019
 #5
avatar+6248 
+2

OMG IT'S CORNER FOR YOU!!!!

 

:D :D :D

Rom  Sep 25, 2019

1 Online Users

avatar